subject
Mathematics, 22.11.2019 02:31 gerardogonzale362

One of the two hypotheses is to be chosen based on an observation of the random variable x. if h0 is true, then x is exponentially distributed with parameter m = 5. if h1 is true, then x is uniformly distributed on the interval [0,1]. (a) find the likelihood ratio k(x) for 0 # x 1 3. (b) for what values of the observation x (for 0 < x 1 3) does the maximum likelihood decision rule declare that h1 is true? (c) find the conditional probability of a false alarm, given that h0 is true. (d) find the conditional probability of a miss, given that h1 is true.

ansver
Answers: 2

Another question on Mathematics

question
Mathematics, 21.06.2019 17:00
Convert 2000 swiss francs to dutch guilders
Answers: 1
question
Mathematics, 21.06.2019 17:30
In parallelogram abcd the ratio of ab to bcis 5: 3. if the perimeter of abcd is 32 find ab
Answers: 1
question
Mathematics, 21.06.2019 17:30
You can find the constant of proportionality by finding the ratio of
Answers: 1
question
Mathematics, 21.06.2019 19:00
Billy plotted −3 4 and −1 4 on a number line to determine that −3 4 is smaller than −1 4 is he correct? explain why or why not?
Answers: 3
You know the right answer?
One of the two hypotheses is to be chosen based on an observation of the random variable x. if h0 is...
Questions
question
History, 03.09.2020 23:01
question
History, 03.09.2020 23:01
question
Mathematics, 03.09.2020 23:01